Fourier transform of the hyperbolic secant function

Click For Summary
The discussion focuses on finding the Fourier transform of the hyperbolic secant function, with the expected result being 4∑((-1)^n(1+2n))/(ω^2(2n+1)^2). The user suggests using the geometric series and integrating the product of the hyperbolic secant and e^(iωt) from -∞ to ∞. A key insight shared is that the term (-1)^n arises from the series expansion of 1/(1+a), which alternates signs. The user expresses gratitude upon understanding this concept, indicating successful clarification of the problem. Overall, the thread highlights the process of deriving the Fourier transform and clarifying the series expansion involved.
Marie_Curie
Messages
3
Reaction score
0

Homework Statement



Hi there!

I'm just trying to figure out the Fourier transform of the hyperbolic secant function... I already know the outcome:

4\sum\ ((-1)^n*(1+2n))/(ω^2*(2n+1)^2)

But sadly, I cannot figure out how to work round to it! :( maybe one of you could help me...

Homework Equations



I was thinking of using the geometric series 1/(1+q) = \sum (-q)^n for q = e^(-2*x), as the hyperbolic secant is 2*e^(-x)/(1+e^(-2*x)) .
And then you need to multiply the hyperbolic secant with e^(iωt) and integrate from -∞
up to ∞.
But... frankly, that was it. I have never managed to come to the result
4\sum\ ((-1)^n*(1+2n))/(ω^2*(2n+1)^2)

Maybe someone could give me a hint to to solve this problem...? :)

Lots of greetings,
Marie
 
Last edited:
Physics news on Phys.org
Marie_Curie said:

Homework Statement



Hi there!

I'm just trying to figure out the Fourier transform of the hyperbolic secant function... I already know the outcome:

4\sum\ ((-1)^n*(1+2n))/(ω^2*(2n+1)^2)

But sadly, I cannot figure out how to work round to it! :( maybe one of you could help me...

Homework Equations



I was thinking of using the geometric series 1/(1+q) = \sum (-q)^n for q = e^(-2*x), as the hyperbolic secant is 2*e^(-x)/(1+e^(-2*x)) .
And then you need to multiply the hyperbolic secant with e^(iωt) and integrate from -∞
up to ∞.
But... frankly, that was it. I have never managed to come to the result
4\sum\ ((-1)^n*(1+2n))/(ω^2*(2n+1)^2)

Maybe someone could give me a hint to to solve this problem...? :)

Lots of greetings,
Marie

f(x) = sech(x) is an even function, so the FT of f is (g + conjugate g), where ##g = \int_0^{\infty} f(x) e^{i \omega x} \, dx.## Apply this term-by-term to
\text{sech}(x) = 2 \sum (-1)^n e^{-(2n+1)x}.
 
Ray Vickson said:
f(x) = sech(x) is an even function, so the FT of f is (g + conjugate g), where ##g = \int_0^{\infty} f(x) e^{i \omega x} \, dx.## Apply this term-by-term to
\text{sech}(x) = 2 \sum (-1)^n e^{-(2n+1)x}.


Aaahh, wonderful! Thank you!

Just one more question... where does the Term (-1)^n come from?

Thanks a lot!
MARIE
 
Marie_Curie said:
Aaahh, wonderful! Thank you!

Just one more question... where does the Term (-1)^n come from?

Thanks a lot!
MARIE

It comes from the expansion of 1/(1+a)=1-a+a^2-a^3+... It's the factor that alternates sign.
 
Last edited:
Dick said:
It comes from the expansion of 1/(1-a)=1-a+a^2-a^3+... It's the factor that alternates sign.

Oh yes, sure! Now I got it :)

Thank you very much! :) And have a nice evening!

_MARIE_
 
Marie_Curie said:
Oh yes, sure! Now I got it :)

Thank you very much! :) And have a nice evening!

_MARIE_

You're welcome. Nice evening to you too. Note I goofed on the post. I meant 1/(1+a) not 1/(1-a).
 
Question: A clock's minute hand has length 4 and its hour hand has length 3. What is the distance between the tips at the moment when it is increasing most rapidly?(Putnam Exam Question) Answer: Making assumption that both the hands moves at constant angular velocities, the answer is ## \sqrt{7} .## But don't you think this assumption is somewhat doubtful and wrong?

Similar threads

  • · Replies 3 ·
Replies
3
Views
2K
Replies
1
Views
2K
  • · Replies 5 ·
Replies
5
Views
2K
  • · Replies 16 ·
Replies
16
Views
2K
  • · Replies 6 ·
Replies
6
Views
3K
  • · Replies 1 ·
Replies
1
Views
2K
  • · Replies 6 ·
Replies
6
Views
1K
  • · Replies 2 ·
Replies
2
Views
1K
  • · Replies 1 ·
Replies
1
Views
2K
Replies
5
Views
2K